DETERMINANT of matrix $M_n$












0












$begingroup$


Let $M_n=
begin{bmatrix}
n-1 & n-2 & n-3 & ... & 1& 0\
n-2 & n-3 & n-4 & ... & 0& 1\
n-3 & n-4 & n-5 & ... & 1&2\
vdots & vdots & vdots & ddots & vdots & vdots\
1& 0 & 1 &...& n-3 & n-2\
0 & 1 & 2 &... & n-2&n-1
end{bmatrix}$



What's $det(M_n)$ ?



Can someone help me with this, I tried Using Laplace's formula and transformations of rows but it didn't work.
original problem










share|cite|improve this question











$endgroup$








  • 1




    $begingroup$
    Looks like a special case of math.stackexchange.com/questions/770117/….
    $endgroup$
    – Martin R
    Jan 18 at 22:07






  • 1




    $begingroup$
    @MartinR Not sure. In this case $a_{11}=a_1=n-1$, while $a_{n2}=1neq a_{11}$. Nevertheless, it may be a OP's typo. We will see...
    $endgroup$
    – Dog_69
    Jan 18 at 22:42








  • 1




    $begingroup$
    @m2017m Please see MartinR's comment and check your formula. There might be a typo in you second and third columns.
    $endgroup$
    – Dog_69
    Jan 18 at 22:46








  • 1




    $begingroup$
    there is no typo, formula is good
    $endgroup$
    – m2017m
    Jan 19 at 11:07
















0












$begingroup$


Let $M_n=
begin{bmatrix}
n-1 & n-2 & n-3 & ... & 1& 0\
n-2 & n-3 & n-4 & ... & 0& 1\
n-3 & n-4 & n-5 & ... & 1&2\
vdots & vdots & vdots & ddots & vdots & vdots\
1& 0 & 1 &...& n-3 & n-2\
0 & 1 & 2 &... & n-2&n-1
end{bmatrix}$



What's $det(M_n)$ ?



Can someone help me with this, I tried Using Laplace's formula and transformations of rows but it didn't work.
original problem










share|cite|improve this question











$endgroup$








  • 1




    $begingroup$
    Looks like a special case of math.stackexchange.com/questions/770117/….
    $endgroup$
    – Martin R
    Jan 18 at 22:07






  • 1




    $begingroup$
    @MartinR Not sure. In this case $a_{11}=a_1=n-1$, while $a_{n2}=1neq a_{11}$. Nevertheless, it may be a OP's typo. We will see...
    $endgroup$
    – Dog_69
    Jan 18 at 22:42








  • 1




    $begingroup$
    @m2017m Please see MartinR's comment and check your formula. There might be a typo in you second and third columns.
    $endgroup$
    – Dog_69
    Jan 18 at 22:46








  • 1




    $begingroup$
    there is no typo, formula is good
    $endgroup$
    – m2017m
    Jan 19 at 11:07














0












0








0





$begingroup$


Let $M_n=
begin{bmatrix}
n-1 & n-2 & n-3 & ... & 1& 0\
n-2 & n-3 & n-4 & ... & 0& 1\
n-3 & n-4 & n-5 & ... & 1&2\
vdots & vdots & vdots & ddots & vdots & vdots\
1& 0 & 1 &...& n-3 & n-2\
0 & 1 & 2 &... & n-2&n-1
end{bmatrix}$



What's $det(M_n)$ ?



Can someone help me with this, I tried Using Laplace's formula and transformations of rows but it didn't work.
original problem










share|cite|improve this question











$endgroup$




Let $M_n=
begin{bmatrix}
n-1 & n-2 & n-3 & ... & 1& 0\
n-2 & n-3 & n-4 & ... & 0& 1\
n-3 & n-4 & n-5 & ... & 1&2\
vdots & vdots & vdots & ddots & vdots & vdots\
1& 0 & 1 &...& n-3 & n-2\
0 & 1 & 2 &... & n-2&n-1
end{bmatrix}$



What's $det(M_n)$ ?



Can someone help me with this, I tried Using Laplace's formula and transformations of rows but it didn't work.
original problem







linear-algebra






share|cite|improve this question















share|cite|improve this question













share|cite|improve this question




share|cite|improve this question








edited Jan 19 at 11:05







m2017m

















asked Jan 18 at 22:04









m2017mm2017m

94




94








  • 1




    $begingroup$
    Looks like a special case of math.stackexchange.com/questions/770117/….
    $endgroup$
    – Martin R
    Jan 18 at 22:07






  • 1




    $begingroup$
    @MartinR Not sure. In this case $a_{11}=a_1=n-1$, while $a_{n2}=1neq a_{11}$. Nevertheless, it may be a OP's typo. We will see...
    $endgroup$
    – Dog_69
    Jan 18 at 22:42








  • 1




    $begingroup$
    @m2017m Please see MartinR's comment and check your formula. There might be a typo in you second and third columns.
    $endgroup$
    – Dog_69
    Jan 18 at 22:46








  • 1




    $begingroup$
    there is no typo, formula is good
    $endgroup$
    – m2017m
    Jan 19 at 11:07














  • 1




    $begingroup$
    Looks like a special case of math.stackexchange.com/questions/770117/….
    $endgroup$
    – Martin R
    Jan 18 at 22:07






  • 1




    $begingroup$
    @MartinR Not sure. In this case $a_{11}=a_1=n-1$, while $a_{n2}=1neq a_{11}$. Nevertheless, it may be a OP's typo. We will see...
    $endgroup$
    – Dog_69
    Jan 18 at 22:42








  • 1




    $begingroup$
    @m2017m Please see MartinR's comment and check your formula. There might be a typo in you second and third columns.
    $endgroup$
    – Dog_69
    Jan 18 at 22:46








  • 1




    $begingroup$
    there is no typo, formula is good
    $endgroup$
    – m2017m
    Jan 19 at 11:07








1




1




$begingroup$
Looks like a special case of math.stackexchange.com/questions/770117/….
$endgroup$
– Martin R
Jan 18 at 22:07




$begingroup$
Looks like a special case of math.stackexchange.com/questions/770117/….
$endgroup$
– Martin R
Jan 18 at 22:07




1




1




$begingroup$
@MartinR Not sure. In this case $a_{11}=a_1=n-1$, while $a_{n2}=1neq a_{11}$. Nevertheless, it may be a OP's typo. We will see...
$endgroup$
– Dog_69
Jan 18 at 22:42






$begingroup$
@MartinR Not sure. In this case $a_{11}=a_1=n-1$, while $a_{n2}=1neq a_{11}$. Nevertheless, it may be a OP's typo. We will see...
$endgroup$
– Dog_69
Jan 18 at 22:42






1




1




$begingroup$
@m2017m Please see MartinR's comment and check your formula. There might be a typo in you second and third columns.
$endgroup$
– Dog_69
Jan 18 at 22:46






$begingroup$
@m2017m Please see MartinR's comment and check your formula. There might be a typo in you second and third columns.
$endgroup$
– Dog_69
Jan 18 at 22:46






1




1




$begingroup$
there is no typo, formula is good
$endgroup$
– m2017m
Jan 19 at 11:07




$begingroup$
there is no typo, formula is good
$endgroup$
– m2017m
Jan 19 at 11:07










0






active

oldest

votes












Your Answer








StackExchange.ready(function() {
var channelOptions = {
tags: "".split(" "),
id: "69"
};
initTagRenderer("".split(" "), "".split(" "), channelOptions);

StackExchange.using("externalEditor", function() {
// Have to fire editor after snippets, if snippets enabled
if (StackExchange.settings.snippets.snippetsEnabled) {
StackExchange.using("snippets", function() {
createEditor();
});
}
else {
createEditor();
}
});

function createEditor() {
StackExchange.prepareEditor({
heartbeatType: 'answer',
autoActivateHeartbeat: false,
convertImagesToLinks: true,
noModals: true,
showLowRepImageUploadWarning: true,
reputationToPostImages: 10,
bindNavPrevention: true,
postfix: "",
imageUploader: {
brandingHtml: "Powered by u003ca class="icon-imgur-white" href="https://imgur.com/"u003eu003c/au003e",
contentPolicyHtml: "User contributions licensed under u003ca href="https://creativecommons.org/licenses/by-sa/3.0/"u003ecc by-sa 3.0 with attribution requiredu003c/au003e u003ca href="https://stackoverflow.com/legal/content-policy"u003e(content policy)u003c/au003e",
allowUrls: true
},
noCode: true, onDemand: true,
discardSelector: ".discard-answer"
,immediatelyShowMarkdownHelp:true
});


}
});














draft saved

draft discarded


















StackExchange.ready(
function () {
StackExchange.openid.initPostLogin('.new-post-login', 'https%3a%2f%2fmath.stackexchange.com%2fquestions%2f3078810%2fdeterminant-of-matrix-m-n%23new-answer', 'question_page');
}
);

Post as a guest















Required, but never shown

























0






active

oldest

votes








0






active

oldest

votes









active

oldest

votes






active

oldest

votes
















draft saved

draft discarded




















































Thanks for contributing an answer to Mathematics Stack Exchange!


  • Please be sure to answer the question. Provide details and share your research!

But avoid



  • Asking for help, clarification, or responding to other answers.

  • Making statements based on opinion; back them up with references or personal experience.


Use MathJax to format equations. MathJax reference.


To learn more, see our tips on writing great answers.




draft saved


draft discarded














StackExchange.ready(
function () {
StackExchange.openid.initPostLogin('.new-post-login', 'https%3a%2f%2fmath.stackexchange.com%2fquestions%2f3078810%2fdeterminant-of-matrix-m-n%23new-answer', 'question_page');
}
);

Post as a guest















Required, but never shown





















































Required, but never shown














Required, but never shown












Required, but never shown







Required, but never shown

































Required, but never shown














Required, but never shown












Required, but never shown







Required, but never shown







Popular posts from this blog

Human spaceflight

Can not write log (Is /dev/pts mounted?) - openpty in Ubuntu-on-Windows?

File:DeusFollowingSea.jpg